Difference between revisions of "2025 SSMO Accuracy Round Problems/Problem 8"
(→Problem) |
(→Solution) |
||
(2 intermediate revisions by the same user not shown) | |||
Line 4: | Line 4: | ||
==Solution== | ==Solution== | ||
+ | |||
+ | Let <math>A = (a_1, a_2, \dots, a_{10})</math> be an arbitrary permutation in <math>\mathcal{S}</math>, let <math>p</math> and <math>q</math> be as in the problem statement, and let <math>1\le r \le 10</math> be the integer such that <math>a_r = 10</math>. | ||
+ | |||
+ | The key to solving this problem is to formulate a way of uniquely constructing every possible <math>A</math>. To do this, we first introduce two bits of intuitive terminology; we say that a term <math>a_x \ne a_r</math> of <math>A</math> is <i>lefty</i> if <math>x<r</math> and <i>righty</i> if <math>x>r</math>. An example showcases these definitions best: | ||
+ | <cmath>A = (\underbrace{2,4,5,6}_{\text{Lefty terms}},10,\underbrace{9,8,7,3,1}_{\text{Righty terms}}).</cmath> | ||
+ | Assign every term of <math>A</math> not equal to <math>10</math> to be either lefty or righty. We claim that this uniquely determines a peaked permutation <math>A</math>; in fact, we show that the lefty terms must be in increasing order and the righty terms must be in decreasing order (and if this is the case, it is easy to verify that <math>A</math> is indeed peaked). For the sake of contradiction, suppose that there exist two lefty terms <math>a_{x_1} > a_{x_2}</math> with <math>x_1 < x_2</math>. However, we then have <math>a_{x_1} > a_{x_2}</math> and <math>a_{x_2} < a_r</math> with <math>1 \le x_1 < x_2 < r \le 10</math>; contradiction. Therefore, the lefty terms are in increasing order. A similar argument proves that the righty terms are in decreasing order. | ||
+ | |||
+ | Let <math>N</math> be the number of indices <math>\ell</math> such that <math>a_\ell</math> lies between <math>a_p = 4</math> and <math>a_q = 9</math> in <math>A</math>; that is, when either <math>p < \ell < q</math> or <math>p > \ell > q</math>. Note that <math>\mathbb{E}[|p-q|] = \mathbb{E}[N] + 1</math>, so we focus on finding <math>\mathbb{E}[N]</math>. We do this via casework on whether <math>4</math> and <math>9</math> lie on the same side of <math>10</math> in <math>A</math> or not. | ||
+ | |||
+ | <i>Case 1:</i> <math>4</math> and <math>9</math> lie on the same side of <math>10</math> (that is, they are both lefty or both righty). Clearly, <math>10</math> does not lie between <math>4</math> and <math>9</math>. Some term <math>d\ne 10</math> can only lie between <math>4</math> and <math>9</math> whenever <math>4 < d < 9</math>; there are <math>4</math> such <math>d</math>. Furthermore, <math>d</math> must lie on the same of <math>10</math> as <math>4</math> and <math>9</math>, which happens with probability <math>\tfrac{1}{2}</math>. By the linearity of expected value, the expected value of <math>N</math> in this case is <math> 4\cdot \tfrac{1}{2} = 2</math>. | ||
+ | |||
+ | <i>Case 2:</i> <math>4</math> and <math>9</math> lie on opposite sides of <math>10</math> (that is, one of them is lefty and the other is righty). Clearly, <math>10</math> lies between <math>4</math> and <math>9</math>. Some term <math>d\ne 10</math> lies between <math>4</math> and <math>9</math> only when <math>4 < d < 10</math> (it is impossible that <math>9 < d < 10</math>); there are <math>4</math> such <math>d</math>. Then, <math>d</math> must lie on on the same side of <math>10</math> as <math>4</math>, which happens with probability <math>\tfrac{1}{2}</math>. The expected value of <math>N</math> in this case is thus <math>1 + 4\cdot \tfrac{1}{2} = 3</math>. | ||
+ | |||
+ | Each of the two cases just outlined occurs with probability <math>\tfrac{1}{2}</math>, so <math>\mathbb{E}[N] = \tfrac{1}{2}\cdot 2 + \tfrac{1}{2}\cdot 3 = \tfrac{5}{2}</math>. The expected value of <math>|p-q|</math> is <math>\tfrac{5}{2}+1 = \tfrac{7}{2}</math>, so the answer is <math>7+2 = \boxed{9}</math>. | ||
+ | |||
+ | ~Sedro |
Latest revision as of 14:02, 18 September 2025
Problem
We say that a permutation of the integers
through
inclusive is peaked if there do not exist three integers
such that
and
. Let
be the set of all peaked permutations. If
and
, the expected value of
over all permutations in
can be written as
, where
and
are relatively prime positive integers. What is the value of
?
Solution
Let be an arbitrary permutation in
, let
and
be as in the problem statement, and let
be the integer such that
.
The key to solving this problem is to formulate a way of uniquely constructing every possible . To do this, we first introduce two bits of intuitive terminology; we say that a term
of
is lefty if
and righty if
. An example showcases these definitions best:
Assign every term of
not equal to
to be either lefty or righty. We claim that this uniquely determines a peaked permutation
; in fact, we show that the lefty terms must be in increasing order and the righty terms must be in decreasing order (and if this is the case, it is easy to verify that
is indeed peaked). For the sake of contradiction, suppose that there exist two lefty terms
with
. However, we then have
and
with
; contradiction. Therefore, the lefty terms are in increasing order. A similar argument proves that the righty terms are in decreasing order.
Let be the number of indices
such that
lies between
and
in
; that is, when either
or
. Note that
, so we focus on finding
. We do this via casework on whether
and
lie on the same side of
in
or not.
Case 1: and
lie on the same side of
(that is, they are both lefty or both righty). Clearly,
does not lie between
and
. Some term
can only lie between
and
whenever
; there are
such
. Furthermore,
must lie on the same of
as
and
, which happens with probability
. By the linearity of expected value, the expected value of
in this case is
.
Case 2: and
lie on opposite sides of
(that is, one of them is lefty and the other is righty). Clearly,
lies between
and
. Some term
lies between
and
only when
(it is impossible that
); there are
such
. Then,
must lie on on the same side of
as
, which happens with probability
. The expected value of
in this case is thus
.
Each of the two cases just outlined occurs with probability , so
. The expected value of
is
, so the answer is
.
~Sedro